Difference between revisions of "2009 AMC 8 Problems/Problem 5"

(Problem)
Line 8: Line 8:
 
\textbf{(D)}\  68  \qquad
 
\textbf{(D)}\  68  \qquad
 
\textbf{(E)}\    99</math>
 
\textbf{(E)}\    99</math>
 +
 +
==Solution==
 +
List them out, adding the three previous numbers to get the next number,
 +
 +
<cmath>1,2,3,6,11,20,37,\boxed{\textbf{(D)}\ 68}</cmath>
  
 
==See Also==
 
==See Also==
 
{{AMC8 box|year=2009|num-b=4|num-a=6}}
 
{{AMC8 box|year=2009|num-b=4|num-a=6}}

Revision as of 16:17, 25 December 2012

Problem

A sequence of numbers starts with $1$, $2$, and $3$. The fourth number of the sequence is the sum of the previous three numbers in the sequence: $1\plus{}2\plus{}3\equal{}6$ (Error compiling LaTeX. Unknown error_msg). In the same way, every number after the fourth is the sum of the previous three numbers. What is the eighth number in the sequence?

$\textbf{(A)}\  11  \qquad \textbf{(B)}\   20  \qquad \textbf{(C)}\   37  \qquad \textbf{(D)}\   68  \qquad \textbf{(E)}\    99$

Solution

List them out, adding the three previous numbers to get the next number,

\[1,2,3,6,11,20,37,\boxed{\textbf{(D)}\ 68}\]

See Also

2009 AMC 8 (ProblemsAnswer KeyResources)
Preceded by
Problem 4
Followed by
Problem 6
1 2 3 4 5 6 7 8 9 10 11 12 13 14 15 16 17 18 19 20 21 22 23 24 25
All AJHSME/AMC 8 Problems and Solutions